0 Daumen
260 Aufrufe

Aufgabe:

Für ein festes q ∈ R, ist die Folge an = n · qn nicht umbedingt monoton. Für q ≥ 0 gibt es aber immer einen Index N = Nq ∈ N, sodass die Folge an monton ist, also dass die Monotonie zumindest ab diesem Index gilt. Bestimme für jedes q ≥ 0 so einen Index N = Nq.

Problem/Ansatz:

ich weiß nicht wirklich wie ich da rann gehen soll. Wäre N ≥ 0 richtig? :)

Avatar von

1 Antwort

0 Daumen

Die Folge wächst, wenn die Differenz a_(n+1) - a_n

positiv ist, und sie fällt, wenn diese Differenz negativ ist.

Werte die Differenz \((n+1)q^{n+1}-nq^n\) dahingehend aus!

Tipp: Klammere den ggT von Minuend und Subtrahend aus.

Avatar von 54 k 🚀

habe n ≥ -1 für wachsend und n ≤ -1 für fallend herausbekommen.

Das ist Unsinn. Ob das Verhalten sehr zeitig oder erst sehr spät wechselt, hängt auch von q ab. Wohin ist das q in deiner Rechnung verschwunden?

Außerdem ist n keine negative Zahl. Was willst du da mit -1?

habs jetzt nochmal gerechnet. Komme auf n ≥ \( \frac{1}{ln(q)} \) und n ≥ \( \frac{-q}{q-1} \)

also n ≥ \( \frac{1}{ln(q)} \) für q ≥ 0? :)

Mal nachrechnen:

\((n+1)q^{n+1}-nq^n=q^n((n+1)q-n)\)

Für q>0 ist q^n ohnehin immer positiv, also hängt das Vorzeichen des Terms nur vom Faktor (n+1)q-n.

Angenommen, dieser Faktor ist positiv:

(n+1)q-n>0

nq+q-n>0

n(q-1)+q>0

n(q-1)>-q

Fall 1: q>1

n>\( \frac{-q}{q-1}=\frac{q}{1-q} \) gilt für alle n.

Fall 2: q=1

0>-1

ist für alle n erfüllt. (Die Folge wäre (1 , 2, 3, ...) und steigt immer.)

Fall 3: 0<q<1

n<\( \frac{-q}{q-1}=\frac{q}{1-q} \)

Ok, dankeschön.

Ein anderes Problem?

Stell deine Frage

Willkommen bei der Mathelounge! Stell deine Frage einfach und kostenlos

x
Made by a lovely community